Đến nội dung

royal1534 nội dung

Có 759 mục bởi royal1534 (Tìm giới hạn từ 09-05-2020)



Sắp theo                Sắp xếp  

#666889 $\sum \dfrac{abc+b+c-a}{a^2+1} \geq...

Đã gửi bởi royal1534 on 04-01-2017 - 01:00 trong Bất đẳng thức và cực trị

Với $a,b,c \geq 0$ chứng minh rằng
$\dfrac{abc+b+c-a}{a^2+1}+\dfrac{abc+c+a-b}{b^2+1} +\dfrac{abc+a+b-c}{c^2+1} \geq a+b+c$

Nice problem ! 

Lời giải :

Để ý rằng $a+b+c=(b+c-a)+(c+a-b)+(a+b-c).$

BĐT cần chứng minh tương đương với : 

$$ \sum [\frac{abc+b+c-a}{a^2+1}-(b+c-a)] \geq 0$$

$$\frac{a(a-b)(a-c)}{a^2+1}+\frac{b(b-c)(b-a)}{b^2+1}+\frac{c(c-a)(c-b)}{c^2+1} \geq 0$$

Giả sử $a \geq b \geq c$. Theo tiêu chuẩn của bđt Vornicu Schur thì ta cần chứng minh :

$$\frac{a^2}{a^2+1} \geq \frac{b^2}{b^2+1} \Leftrightarrow a \geq b$$ Đúng theo điều đã giả sử.

Chứng minh hoàn tất.




#666746 $2.3^{x}+1=7.5^{y}$

Đã gửi bởi royal1534 on 03-01-2017 - 02:10 trong Số học

Tìm nghiệm nguyên không âm của PT:

                                                                     $2.3^{x}+1=7.5^{y}$

Xét $x=0 \Rightarrow 3=7.5^y$ (Loại).

Xét $y=0 \Rightarrow 2.3^x=6 \Rightarrow x=1$ (thỏa)

Xét $x,y \geq 1$

Ta có $VT \equiv 1 \equiv VP \equiv (-1)^y$  ($mod 3$)

$\Rightarrow y$ chẵn.

Tương tự dễ chứng minh được $x$ lẻ.

 

Đặt $x=2k+1,y=2t$ thì phương trình đã cho tương đương với :

$6.(3^k)^2+1=7.(5^t)^2$

Đặt tiếp $3^k=a, 5^t=b$

$\Rightarrow 7b^2-6a^2=1$

Đây là phương trình Pell dạng $Ax^2-By^2=1$. Nghiệm của phương trình này được sinh ra bởi dãy :

$\left\{\begin{matrix} b_{0}=1,b_{1}=25,b_{n+2}=26b_{n+1}-b_{n} & \\   a_{0}=1,a_{1}=27,a_{n+2}=26a_{n+1}-a_{n}\\n=0,1,..\end{matrix}\right.$
Với $k=1,2$ thì $a=3,9$ (PT vô nghiệm).
Với $k=3$ thì $t=2$. PT có nghiệm $(x,y)=(7,4)$
Với $k \geq 4$.
Gọi $r_{n}$ là số dư khi chia $a_{n}$ cho $81$
Bằng cách chật vật tính toán. Ta tính được :
$r_{0}=1,r_{1}=27,r_{2}=53,r_{3}=55,r_{4}=0,r_{5}=26,r_{6}=28,r_{7}=54,r_{8}=80,r_{9}=1,r_{10}=27.$
Do đó dãy $r_{n}$ tuần hoàn chu kỳ $9$ và ta có $r_{n} \equiv 0 (mod 81) \Leftrightarrow n \equiv 4 (mod 9)$
Tương tự. Gọi $s_{n}$ là số dư khi chia $a_{n}$ cho $19$.
Tương tự ta cũng có $s_{n} \equiv 0 (mod 19) \Leftrightarrow n \equiv 4 (mod 9)$
Từ đó nếu $81 \mid a_{n}$ thì $19 \mid a_{n}$ (Vô lý vì $a_{n}$ có dạng $3^k$).
Vậy với $k \geq 4$ thì phương trình vô nghiệm
Kết luận : $(x,y)=(7,4);(1,0)$




#665971 $2^x+7^y=3^z$

Đã gửi bởi royal1534 on 26-12-2016 - 23:48 trong Số học

Bài toán : Tìm nghiệm nguyên không âm của phương trình sau :

$$2^x+7^y=3^z$$




#665781 $\sum \frac{a^{3}}{a^{2}+b^...

Đã gửi bởi royal1534 on 25-12-2016 - 08:50 trong Bất đẳng thức - Cực trị

Cho a, b, c > 0. Chứng minh rằng:

$\sum \frac{a^{3}}{a^{2}+b^{2}+6ab}\geq \frac{a+b+c}{8}$

Nguyễn Phúc Tăng

Áp dụng bất đẳng thức Holder ta có :

$\sum \frac{a^3}{a^2+b^2+6ab} \geq \frac{(a+b+c)^3}{6(\sum a^2+\sum 3ab)} \geq \frac{(a+b+c)^3}{8(a+b+c)^2}=\frac{a+b+c}{8}$

Q.E.D...




#664988 $\frac{1}{x}+\frac{1}{y...

Đã gửi bởi royal1534 on 18-12-2016 - 12:36 trong Số học

Có nhầm không nhỉ?

Nếu $a_{1}=2$ thì suy ra n là số chính phương chứ? 

Nhầm lẫn tí. Đã fix :D 




#664943 Cho $a,b,c>0$ thỏa mãn $abc=1$. CMR: $\sum...

Đã gửi bởi royal1534 on 18-12-2016 - 01:21 trong Bất đẳng thức - Cực trị

Cho $a,b,c>0$ thỏa mãn $abc=1$. CMR:

$a)$ $\sum\frac{1}{a^5(b+2c)^2}\geq\frac{1}{3}$

 

Tham khảo ở đây




#664942 $x^2+y^2+z^2=xy+yz+zx+1$

Đã gửi bởi royal1534 on 18-12-2016 - 01:19 trong Số học

Tìm x,y,z nguyên dương thoả mãn $\left\{\begin{matrix} x+y+z=p\\ x^2+y^2+z^2=xy+yz+zx+1 \end{matrix}\right.$, với p là số nguyên tố.

Bài này hình như được trích ra từ bài toán này :D . Tác giả là ông Titu thì phải 

Giải phương trình nghiệm nguyên: $x^3+y^3+z^3-3xyz=p$ (Với $p$ là một số nguyên tố).




#664939 $\frac{1}{x}+\frac{1}{y...

Đã gửi bởi royal1534 on 18-12-2016 - 01:11 trong Số học

Với mỗi số nguyên dương $n$, gọi $S(n)$ là số các cặp sắp thứ tự $(x;y)$ nguyên dương thoả mãn $\frac{1}{x}+\frac{1}{y}=\frac{1}{n}$. Hãy tìm các số nguyên dương $n$ sao cho $S(n)=5$

Phương trình đã cho tương đương với :

$$(x-n)(y-n)=n^2$$.

Giả sử $n=p_{1}^{a_{1}}.p_{2}^{a_{2}}...p_{n}^{a_{n}}$.

Thay vào lại phương trình : $$(x-n)(y-n)=p_{1}^{2a_{1}}.p_{2}^{2a_{2}}...p_{n}^{2a_{n}}$$

Nhận xét : Phương trình $ab=n$ có số nghiệm là số ước của $n$.

Mặt khác ta có số ước của $n^2$ được tính bằng : $$(2a_{1}+1)(2a_{2}+1)...(2a_{n}+1)$$

$$\Rightarrow (2a_{1}+1)(2a_{2}+1)...(2a_{n}+1)=5$$

$$\Leftrightarrow a_{1}=2, a_{2}=a_{3}=...=a_{n}=0$$
Vậy thì ta có $n=p^2$ với $p$ là một số nguyên tố thì thỏa mãn điều kiện đề bài. 




#664430 $a+b+c=1$ tim GTLN $ P=(a-b)(b-c)(c-a)$

Đã gửi bởi royal1534 on 12-12-2016 - 00:00 trong Bất đẳng thức và cực trị

$a+b+c=1$ tim GTLN $ P=(a-b)(b-c)(c-a)$

Lời giải :

Xét $P^2=(a-b)^2(b-c)^2(c-a)^2.$
Không mất tính tổng quát. Giả sử $a \geq b \geq c.$
Ta suy ra các đánh giá sau:
$(b-c)^2 \leq b^2, (c-a)^2 \leq a^2$
Suy ra $P^2 \leq a^2b^2(a-b)^2$
Áp dụng bđt Cauchy ta có :
$4P^2=2ab.2ab.(a-b)^2 \leq \frac{[2ab+2ab+(a-b)^2]^3}{27}=\frac{(a+b)^6}{27} \leq \frac{(a+b+c)^6}{27}=\frac{1}{27}$
$\Rightarrow P^2 \leq \frac{1}{27.4}=\frac{1}{108}$
$\Rightarrow P \leq \frac{1}{6\sqrt{3}}$
Đẳng thức xảy ra khi $(a,b,c)=(\frac{3+\sqrt{3}}{6},\frac{3-\sqrt{3}}{6},0)$




#664423 Chứng minh $\sum \frac{a^3}{b^2+c^2} \geq \frac{3}{2...

Đã gửi bởi royal1534 on 11-12-2016 - 22:50 trong Bất đẳng thức và cực trị

Cho $a+b+c=3$

CMR $\sum \frac{a^{3}}{b^{2}+c^{2}}\geq \frac{3}{2}$

Mình nghỉ đề phải cho $a,b,c$ dương :)

 

Một lời giải khác :

Vì bất đẳng thức đã cho đối xứng. Giả sử $a \geq b \geq c$

Ta có $(a^3,b^3,c^3)$ và $(b^2+c^2,c^2+a^2,a^2+b^2)$ là hai bộ đơn điệu ngược chiều. Áp dụng bđt Chevbyshev ta có :

$VT=\sum \frac{a^3}{b^2+c^2} \geq \frac{3}{2}. \frac{a^3+b^3+c^3}{a^2+b^2+c^2}$

Ta cần chứng minh : $a^3+b^3+c^3 \geq a^2+b^2+c^2$

Áp dụng bđt Cauchy-Schwarz ta có : $(a^3+b^3+c^3)(a+b+c) \geq (a^2+b^2+c^2)^2$

$\Leftrightarrow 3(a^3+b^3+c^3) \geq (a^2+b^2+c^2)^2 \geq \frac{(a+b+c)^2}{3}.(a^2+b^2+c^2)$

$\Leftrightarrow a^3+b^3+c^3 \geq a^2+b^2+c^2$ (Q.E.D)

BĐT được chứng minh. Đẳng thức xảy ra khi $a=b=c=1$




#662871 Chứng minh $X,Y,Z,T$ đồng viên

Đã gửi bởi royal1534 on 24-11-2016 - 01:11 trong Hình học

Bài toán: Cho tam giác $ABC$. $D$ là một điểm trên cạnh $BC$. Đường tròn ngoại tiếp tam giác $ABD$ cắt $AC$ tại điểm thứ $2$ là $E$. Đường tròn ngoại tiếp tam giác $ACD$ cắt $AB$ tại điểm thứ hai là $F$. Hai đường tròn $(O_{1}),(O_{2})$ qua hai điểm $A,E$ tiếp xúc $BC$ tại $M,N$. $X$ là giao điểm $ME$ và $NF$. $Y$ là giao điểm $MF$ và $NE$. Hai đường tròn $(O'_{1}).(O'_{2})$ qua hai điểm $A,F$ tiếp xúc $BC$ tại $P,Q$. $Z$ là giao điểm $PE$ và $QF$, $T$ là giao điểm $PF$ và $QE.$

Chứng minh $X,Y,Z,T$ đồng viên 




#661909 Chứng minh ba đường tròn $(ABC)$,$(CDF)$ và $(BDE)...

Đã gửi bởi royal1534 on 14-11-2016 - 19:36 trong Hình học

1.Hai đường tròn $S_{1},S_{2}$ có điểm chung $A$ (Không cần thiết phải tiếp xúc nhau). Qua A vẽ đường thẳng cắt $S_{1}$ tại B và $S_{2}$ tại $C$. Tiếp tuyến tại $B$ và $C$ của hai đường tròn cắt nhau tại $D$. Chứng minh $\widehat{BDC}$ không phụ thuộc vào đường thẳng qua $A$

2.Trên cạnh $AB$ của tứ giác lồi $ABCD$ lấy một điểm $E$ khác $A,B$. Các đoạn $AC$ và $DE$ cắt nhau tại $F$. Chứng minh ba đường tròn $(ABC)$,$(CDF)$ và $(BDE)$ có ít nhất một điểm chung 




#660140 Tìm n là số nguyên dương sao cho $2^{n}+1\vdots n^{2...

Đã gửi bởi royal1534 on 01-11-2016 - 02:16 trong Số học

Chỗ này sai nhé bạn

$(p-1,2n) $ có thể khác 2

Ví dụ $p=5, n=60 $ 

p là ước nguyên tố nhỏ nhất của n ... 




#659916 Tìm p nguyên tố sao cho $p^{3}+\frac{p-1}{...

Đã gửi bởi royal1534 on 30-10-2016 - 08:59 trong Số học

Tìm p nguyên tố sao cho $p^{3}+\frac{p-1}{2}$ là tích 3 số nguyên liên tiếp

Rõ ràng các số nguyên khác 0.

Vì tích 3 số nguyên liên tiếp khác 0 luôn chia hết cho 6.

Ta có $VT=p^3-p+\frac{3p-1}{2}$ chia hết cho $6$ dẫn đến $\frac{3p-1}{2}$ chia hết cho $6$.

Xét $p=4k+1 \Rightarrow \frac{3p-1}{2}=6k+1$ (Loại).

Xét $p=4k+3 \Rightarrow \frac{3p-1}{2}=6k+4$ (Loại).

Vậy không có số nguyên tố p thỏa đề.




#659909 Tìm n là số nguyên dương sao cho $2^{n}+1\vdots n^{2...

Đã gửi bởi royal1534 on 30-10-2016 - 07:58 trong Số học

Tìm n là số nguyên dương sao cho $2^{n}+1\vdots n^{2}$

Bài này trong IMO 1990 :)
Lời giải : 

Xét $n=1$ (thỏa)

 

Xét $n>1$ :

Trước hết ta chứng minh $3 \mid n$

Từ giả thiết ta có : $n \mid 2^n+1 \Leftrightarrow 2^{2n} \equiv 1$ ($mod$ n)

Gợi $p$ là ước nguyên tố nhỏ nhất của $n$ và $h=ord_{n}(2)$

$\Rightarrow$ $2^{2n} \equiv 1$ ($mod$ $p$) và $2^h \equiv 1$ ($mod$ $p$)

Theo định lý Fermat nhỏ ta cũng có : $2^{p-1} \equiv 1 $($mod$ $p$)

$\Rightarrow h \mid p-1 , h \mid 2n$

$\Rightarrow h \mid (p-1,2n) = 2$ (Vì $p-1<p \mid n$ nên $(p-1,n)=1$)

$\Rightarrow h=2$ (Dễ thấy $h=1$ không thỏa)

Từ đó ta có $p \mid 2^2-1=3 \Rightarrow 3 \mid n$

---------------------------------

Đặt $n=3^k.y ($Với $k,y \in N, (y,3)=1 )$

Từ giả thiết ta có : $v_{3}(n^2) \leq v_{3}(2^n+1)$

$\Leftrightarrow 2k \leq k+1$ 

$\Leftrightarrow k=1$

Suy ra $n=3y$ (với $i \in N, (i,3)=1)$

Giả sử $y>1$. Gọi $q$ là một ước nguyên tố của $y$ (q khác 3)

Tương tự như trên suy ra $q \mid 2^{(2n,q-1)}-1 \mid 2^6-1=63 \rightarrow q=7$

Dẫn tới $7 \mid 2^n+1$ (Dễ thấy điều này vô lý vì $VP \equiv 1,2,4 (mod 7)$)

Vậy $y=1 \rightarrow n=3 $

Vậy $n=3,n=1$ thỏa đề




#657791 CHUYÊN ĐỀ: Hàm số và thuần nhất, chuẩn hóa trong chứng minh BĐT-CT THCS

Đã gửi bởi royal1534 on 13-10-2016 - 23:04 trong Chuyên đề toán THCS

Đây là bản pdf của chuyên đề này do mình tự viêt Latex 

Vì mới tập tành latex nên mọi người thông cảm không tránh khỏi những sai sót :D. Hi vọng nó hữu dụng với mọi người. 

P/s: Chuyên File gửi kèm  HSBN.pdf   166.56K   222 Số lần tảiđề hay. Nếu thằng Kiên không vừa lòng cái gì thì inb mình nhé.




#657719 Cần xử lí thành viên này.

Đã gửi bởi royal1534 on 13-10-2016 - 08:33 trong Xử lí vi phạm - Tranh chấp - Khiếu nại

BQT ban thành viên "duc113" dùm em.
Lý do: spam trong box đại số THCS



#657584 $\left | \sum \frac{a^3-b^3}{a+b}...

Đã gửi bởi royal1534 on 12-10-2016 - 01:42 trong Bất đẳng thức và cực trị

Cho a,b,c là các số thực dương. Chứng minh rằng $\left | \frac{a^3-b^3}{a+b}+\frac{b^3-c^3}{b+c}+\frac{c^3-a^3}{c+a} \right |\leq \frac{1}{4}((a-b)^2+(b-c)^2+(c-a)^2)$

Gợi ý : Vì BĐT trên đối xứng. Giả sử $a \geq b \geq c \geq 0$.

Sử dụng BĐT phụ sau: $\frac{a^2+ab+b^2}{a+b} \leq a+\frac{b}{2}$ Ta có đpcm.




#657581 Topic: [LTDH] Mỗi ngày hai bất đẳng thức.

Đã gửi bởi royal1534 on 12-10-2016 - 01:13 trong Bất đẳng thức và cực trị

Tiếp theo: 

Bài 89: Cho $x,y,z$ là ba số thực dương có tổng bằng 3. Tìm GTNN của biểu thức:

$P=3(x^2+y^2+z^2)-2xyz$.Lâu rồi không vào topic của anh :D 

Lâu rồi không vào topic ủng hộ anh :D

-------

Áp dụng bđt AM-GM ta có: $VT \geq (x+y+z)^2-\frac{2(x+y+z)^3}{27}=7$

Vậy $MinP=7$. Dấu '=' xảy ra khi $x=y=z=1$

 

P/s: Sai đề chăng :D. Nếu đổi thành $+2xyz$ thì hay hơn  :icon6: 




#656763 Chứng minh $(DBC)$ tiếp xúc $(AMN)$

Đã gửi bởi royal1534 on 05-10-2016 - 13:41 trong Hình học

Bài toán: Cho $\Delta ABC$ nội tiếp $(O)$. M là trung điểm BC, $I$ là trung điểm cung $BC$ chứa $A$, $(I,IM)$ cắt $AC$ và $AB$ tại $M,N$. Phân giác $\widehat{ABC}$ cắt $(ANM)$ tại $D$. Chứng minh đường tròn ngoại tiếp $\Delta DBC$ tiếp xúc đường tròn ngoại tiếp $\Delta AMN$




#654123 Thăm dò ý kiến về việc thi trắc nghiệm môn toán

Đã gửi bởi royal1534 on 14-09-2016 - 12:02 trong Tin tức - Vấn đề - Sự kiện

Không hiểu Bộ nghĩ cái gì mà cho Toán thi trắc nghiệm nữa. Ngay cả nhiều học sinh,chuyên gia và cả hội toán học phản đối mà vẫn làm thì đúng là giáo dục kiểu bảo thủ thật ._.

P/s: Lớp 10 mà lo quá 




#653065 $I$ là tâm đường tròn nội tiếp tam giác $ABE$.

Đã gửi bởi royal1534 on 06-09-2016 - 21:35 trong Hình học

Bài toán: Cho tam giác $ABC$ cân tại $A$ có $H,M$ lần lượt là trung điểm của $BC,AC$. Đường tròn ngoại tiếp tam giác $BCM$ cắt đoạn $AH$ tại $D$ và đường tròn ngoại tiếp tam giác $ABD$ cắt đoạn $BM$ tại $K$. Gọi $I$ là giao điểm của $AK$ và $BD$, $E$ là giao điểm của $CI$ với $BM$. Chứng minh:
1.Tam giác $AKC$ vuông.
2.$I$ là tâm đường tròn nội tiếp tam giác $ABE$.




#652558 Topic về phương trình và hệ phương trình

Đã gửi bởi royal1534 on 03-09-2016 - 11:02 trong Phương trình - hệ phương trình - bất phương trình

Rút ra/học từ các bài của NTA1907  đó em! Anh không biết tài liệu như vậy!

Vậy chắc phải tự ghi chép lại cho cẩn thận rồi :D 




#652554 Topic về phương trình và hệ phương trình

Đã gửi bởi royal1534 on 03-09-2016 - 10:52 trong Phương trình - hệ phương trình - bất phương trình

 

Vài bài liên tiếp cứ theo tư duy

$$\begin{cases} & u+v= f(u,v),\\ & u^2+v^2=g(u,v,x,y), \end{cases}$$

trong đó $[f(u,v,x,y)]^2=g(u,vx,y)+2uv$ có các nhân tử.

Vài đặc điểm của $f, g$: $f$ thường chứa các số hạng bậc nhất của $u, v$, và $g$ chứa các số hạng bậc hai theo hai biến $ u, v$ (có thể không tường minh theo $u, v$).

 

Cách chọn $u (, v)$: nhìn vào 2 phương trình, trong một phương trình này số hạng $u (,v)$ được xuất hiện, trong phương trình còn lại chứa $u^2 (, v^2)$.

 

 

 

Quào. Những tư duy, kinh nghiệm như thế này được anh tích lũy trong lúc làm bài hả ?. Có tài liệu nào viết về những kiểu như này không ạ.
------ Nếu có thì anh có thể share cho em được không :D




#652315 Topic về phương trình và hệ phương trình

Đã gửi bởi royal1534 on 01-09-2016 - 23:28 trong Phương trình - hệ phương trình - bất phương trình

 

Bài 500: $\left\{\begin{matrix} &(x-2y)\left ( 3x+8y+4\sqrt{x^{2}-4xy+4y^{2}-16} \right )=-6 \\ &(y-4x)\left ( 3y+2x+2\sqrt{x^{2}-4xy+4y^{2}-16} \right )=-10 \end{matrix}\right.$

Bài đã có lời giải ở đây. Nhìn lời giải khủng thật ~.~